№ 10.1. a) Доказать неравенство
\((1) \quad 1+\frac{1}{\sqrt{2}}+\frac{1}{\sqrt{3}}+...+\frac{1}{\sqrt{n}}>\sqrt{n} \quad (n \geq 2) \).

Решение. Прежде чем приступить к доказательству данного неравенства покажем следующее свойство действительных чисел.

Пусть \(x\) и \(y\) - два действительных числа, таких, что \(0<x<y\). Тогда \(\sqrt{x}<\sqrt{y}\).

Действительно,

\( 0<x<y \Leftrightarrow 0<\frac{x}{y}<1 \Leftrightarrow 0<\sqrt{\frac{x}{y}}<1 \Leftrightarrow 0<\sqrt{x}<\sqrt{y} \).

Что и требовалось показать.

Для доказательства неравенства (1) воспользуемся методом математической индукции. Покажем, что при \(n=2\) неравенство (1) верно.

\(1+\frac{1}{\sqrt{2}}=\frac{\sqrt{2}+1}{\sqrt{2}}>\frac{1+1}{\sqrt{2}}=\sqrt{2}\),

\(1+\frac{1}{\sqrt{2}}>\sqrt{2}\).

Что и требовалось показать.

Теперь предположим, что неравенство (1) верно при \(n=k\):

\(1+\frac{1}{\sqrt{2}}+\frac{1}{\sqrt{3}}+...+\frac{1}{\sqrt{k}}>\sqrt{k}\)

и покажем, что оно также верно при \(n=k+1\):
\(\begin{multline}
1+\frac{1}{\sqrt{2}}+\frac{1}{\sqrt{3}}+...+\frac{1}{\sqrt{k}}+\frac{1}{\sqrt{k+1}}> \sqrt{k}+\frac{1}{k+1}=\\
\frac{\sqrt{k}\sqrt{k+1}+1}{\sqrt{k+1}}>\frac{\sqrt{k}\sqrt{k}+1}{\sqrt{k+1}}=\frac{k+1}{\sqrt{k+1}}=\sqrt{k+1}.
\end{multline}\)
Здесь последнее неравенство вытекает из доказанного выше свойства действительных чисел.

Неравенство (1) доказано.